Sunteți pe pagina 1din 12

PHY4604 Fall 2006 R. D.

Field

PHY 4604 Exam 1 Solutions


(Total Points = 100)
Problem 1 (20 points): Circle true or false for following (2 point each).

(a) (True or False) One of the “breakthroughs” that lead to quantum mechanics was the idea of
associating differential operators with the dynamical variables.

(b) (True or False) Solutions of Schrödinger’s equation of the form Ψ ( x, t ) = ψ ( x)φ (t )


correspond to states with definite energy E.

(c) (True or False) Solutions of Schrödinger’s equation of the form Ψ ( x, t ) = ψ ( x)φ (t )


correspond to states in which the probability density ρ ( x, t ) =| Ψ ( x, t ) |2 is independent of time.

(d) (True or False) The wave function Ψ(x,t) must vanish in a region of infinite potential.

(e) (True or False) It is possible for a free particle to have a definite energy.

(f) (True or False) In quantum mechanics particles can enter the “classically forbidden” region
where V0 > E (i.e. KE < 0).

(g) (True or False) The operator AopA↑op is hermitian.

(h) (True or False) If Aop and Bop are hermitian then AopBop is also hermitian.

(i) (True or False) The commutator operator [(px)op,(x2)op] is equal to − 2ih .

(j) (True or False) In position-space the commutator operator [(px)op,sin(kx)] is equal to


− ihk cos(kx) .

Infinite Square Well


Problem 2 (30 points): Consider an electron with mass me confined
within an infinite square well defined by V = +infinity V = +infinity

V(x) = 0 for 0 < x < L,


V(x) = +∞ otherwise.
(a) (2 points) Using Schrödinger’s equation calculate the allowed
stationary state eigenfunctions ψn(x), where the complete wavefunctions 0 L x

are given by Ψn ( x, t ) = ψ n ( x )e n , and normalize the eigenfunctions


− iE t / h

Department of Physics Page 1 of 12 Exam 1 Solutions


PHY4604 Fall 2006 R. D. Field

so that the probability of finding the electron somewhere in the box is one.
2
Answer: ψ n ( x) = sin( nπx / L)
L
Solution: For the region outside of 0 < x < L ψ ( x) = 0 and inside the region
h 2 d 2ψ ( x) d 2ψ ( x) h 2k 2
− = Eψ ( x ) or = − k 2
ψ ( x ) with E =
2me d 2 x d 2x 2me
The most general solution is of the form
ψ ( x) = A sin(kx) + B cos(kx) .
The boundary condition at x = 0 givesψ (0) = B = 0 and the boundary condition at x = L gives
ψ ( L) = A sin(kL) = 0 which implies that kL = nπ with n = 1, 2, 3,… Thus,
n 2π 2h 2
ψ n ( x) = A sin(nπx / L) with En = . The normalization is arrived at by requiring that
2mL2
+∞ L nπ nπ
LA2 LA2  y sin(2 y )  LA2
∫ψ ( x)ψ n ( x)dx = 1 = A ∫0 sin (nπx / L)dx = nπ ∫0 sin ( y)dy = nπ  2 − 4  0 = 2
∗ 2 2 2
n
−∞

Thus, A = 2 / L . These states are called stationary because the probability density and all the
expectation values are independent of time.

(b) (2 points) Show that the wavefunctions Ψn(x,t) correspond to states with definite energy (i.e.
show that ∆E = 0).
Solution: We see that
+∞ +∞ +∞
∂Ψ ( x, t )
< E > n = ∫ Ψn∗ ( x, t ) Eop Ψn ( x, t )dx = ih ∫ Ψn∗ ( x, t ) n dx = En ∫ Ψn∗ ( x, t )Ψn ( x, t )dx = En
−∞ −∞
∂t −∞
and
+∞ +∞ +∞
∂ 2 Ψn ( x, t )
< E 2 > n = ∫ Ψn∗ ( x, t ) Eop2 Ψn ( x, t )dx = −h 2 ∫ Ψn∗ ( x, t ) dx = E n ∫ Ψn ( x, t ) Ψn ( x, t ) dx = En .
2 ∗ 2

−∞ −∞
∂t 2
−∞
Hence
( ∆E ) n = < E 2 > n − ( < E > n ) 2 = 0 .
(c) (2 points) Calculate the allowed energy levels, En, of the system. Express your answer in
terms of the Compton wavelength of the electron, D e = h /( me c ) , and the rest mass energy of the
electron, mec 2 . What is the numerical value of the ground state energy (in MeV) for the case
L = D e ? (Note that mec2 = 0.511 MeV.)
2
n 2π 2  D e  π2
Answers: En =   e ( m c 2
) , E0 = E1 = (me c 2 ) ≈ 2.52MeV
2  L 2
Solution: From part (a) we see that k = nπ/L and hence
2
h 2 k 2 n 2π 2h 2 π 2h 2 π 2  D e 
En = = 2
= n 2
E0 with E0 = 2
= 2
  (mec ) .
2m 2mL 2mL 2  L
The ground state corresponds to n = 1 and hence

Department of Physics Page 2 of 12 Exam 1 Solutions


PHY4604 Fall 2006 R. D. Field

2
π 2  De  π2 π2
E0 = E1 =   (me c ) L
2
→ (me c 2 ) =
=D e
(0.511MeV ) ≈ 2.52MeV .
2  L 2 2
(d) (2 points) Show that the states, ψ n (x) , form an orthonormal set. Namely, show that
+∞

∫ψ ( x)ψ n ( x)dx = δ mn .

m
−∞
Solution:
+∞ L π
2 2
∫ψ m ( x)ψ n ( x)dx =
∫ sin( mπx / L) sin( nπx / L)dx = ∫ sin( my ) sin( my )dy = δ mn

−∞
L0 π 0
th
(e) (3 points) Calculate <x> for n state stationary state.
L
Answer: < x > n =
2
Solution:
+∞ L 2 nπ
2 2 L 
< x > n = ∫ψ ( x)( x) opψ n ( x)dx = ∫ x sin 2 (nπx / L)dx = 

 ∫ y sin
2
( y )dy
L  nπ 
n
−∞
L0 0

2 L  y 2 y sin(2 y ) cos(2 y )  L
=  −
2 
−  =
(nπ )  4 4 8 0 2

(f) (3 points) Calculate <px> for nth state stationary state.


Answer: < px > n = 0
Solution:
+∞ +∞
 ∂ 
< p x > n = ∫ψ ( x)( p x )opψ n ( x)dx =

n ∫ψ

n ( x) − ih ψ n ( x)dx
−∞ −∞  ∂x 

2 nπ
L
2
= −ih
L L 0∫ sin( nπx / L) cos(nπx / L)dx = −ih ∫ sin( y ) cos( y )dy
L 0

2  sin 2 ( y ) 
= −ih   =0
L  2  0
Note that
d < x >n
< px > n = m
.
dt
(g) (5 points) Suppose the electron in this infinite square well has an initial wave function at t =
0 which is an equal mixture of the first two stationary states:
Ψ ( x,0) = A[ψ 1 ( x) + ψ 2 ( x)] .
What is the normalization A? If you measure the energy of this particle, what are the possible
values you might get, and what is the probability of getting each of them? What is the
expectation value of the energy for this state (i.e. average energy)?
Answers: A = 1 / 2 , E1 with probability ½ and energy E2 with probability ½,
5 5π 2h 2
< E >= E0 =
2 4mL2

Department of Physics Page 3 of 12 Exam 1 Solutions


PHY4604 Fall 2006 R. D. Field

Solution: The normalization is arrived at by requiring that


+∞ +∞
∗ 2 ∗ ∗
[ ]
∫ Ψ ( x,0)Ψ ( x,0)dx = 1 = A ∫ ψ 1 ( x) +ψ 2 ( x) [ψ 1 ( x) +ψ 2 ( x)]dx
−∞ −∞
+∞
( )
= A2 ∫ ψ 1∗ ( x)ψ 1 ( x) + ψ 1∗ ( x)ψ 2 ( x) + ψ 2∗ ( x)ψ 1 ( x) + ψ 2∗ ( x)ψ 2 ( x) dx = 2 A2
−∞

and A = 1 / 2 . Thus,
Ψ ( x, t ) = c1ψ 1 ( x)e − iE1t / h + c2ψ 2 ( x)e − iE 2 t / h ,
where c1 = c2 = 1 / 2 which means that you get energy E1 with probability ½ and energy E2 with
probability ½. The expectation value of E is
1 5 5π 2h 2
< E >= 2 E1 + 2 E2 = (E0 + 4 E0 ) = E0 =
1 1
2 2 4mL2
(h) (5 points) Suppose as in part (g) the electron in this infinite square well has a wave function
at t = 0 which is an equal mixture of the first two stationary states as follows
Ψ ( x,0) = A[ψ 1 ( x) + ψ 2 ( x)] .
What is the probability density ρ ( x, t ) =| Ψ ( x, t ) |2 for this state. Does it depend on time? What is
<x> and <px> for this state. Do they depend on time?
L 32  8h ( E − E1 ) 3π 2h
Answers: < x >= 1 + 2 cos(∆ωt )  , < px >= − sin(∆ωt ) , where ∆ω = 2 =
2  9π  3L h 2mL2
Solution: From part (g) we have
1 1
Ψ ( x, t ) = ψ 1 ( x ) e − i ω1 t + ψ 2 ( x ) e − iω 2 t ,
2 2
where ωn = En / h . We see that
ρ ( x, t ) =| Ψ ( x, t ) |2 = 12 | ψ 1 ( x) |2 + 12 | ψ 1 ( x) |2 +ψ 1 ( x)ψ 2 ( x) Re(e −i (ω 2 −ω1 ) t
)
= 12 | ψ 1 ( x) |2 + 12 | ψ 2 ( x) |2 +ψ 1 ( x)ψ 2 ( x) cos(∆ωt )
where
( E2 − E1 ) 3π 2h
∆ω =
= .
h 2mL2
We see that ρ(x,t) depends on time. Also, we see that
+∞ +∞ +∞
< x >= ∫ Ψ ∗ ( x, t )( x)op Ψ ( x, t )dx = 1
2 ∫ψ 1 ( x) xψ 1 ( x)dx + 12 ∫ψ 1 ( x) xψ 1 ( x)dx
∗ ∗

−∞ −∞ −∞
+∞
+ cos(∆ωt ) ∫ xψ 1 ( x)ψ 2 ( x)dx
−∞
+∞
= 12 < x >1 + 12 < x > 2 + cos(∆ωt ) ∫ xψ 1 ( x)ψ 2 ( x)dx
−∞

L L  16 L  L 32 
= + +  2  cos(∆ωt ) = 1 + 2 cos(∆ωt ) 
4 4  9π  2  9π 
where I used

Department of Physics Page 4 of 12 Exam 1 Solutions


PHY4604 Fall 2006 R. D. Field

+∞ L
2
−∞
∫ xψ 1 ( x)ψ 2 ( x)dx = L ∫0
x sin(πx / L) sin(2πx / L)dx

L
1
= ∫ x(cos(3πx / L) − cos(πx / L) )dx
L0
L L
1 1 2 L  1  16 L
=
L0∫ x cos(3πx / L)dx − ∫ x cos(πx / L)dx = 2 1 −  = 2
L0 π  9  9π
and
L 2 3π
1 1 L  L 3π − 2L
L0∫ x cos(3πx / L)dx =  
L  3π  ∫ y cos( y)dy = (3π )
0
2
(cos y + y sin y ) 0 =
(3π ) 2
,

L 2π
1 1L L π − 2L
L0∫ x cos(πx / L)dx =   ∫ y cos( y )dy = 2 (cos y + y sin y ) 0 = 2 .
L π  0 π π
The easiest way to calculate <px> is as follows:
d < x > mL d  32  16mL
< p x >= m = 1 + 2 cos(∆ωt )  = − ∆ω sin( ∆ωt )
dt 2 dt  9π  9π 2
16mL 3π 2h 8h
=− sin( ∆ωt ) = − sin( ∆ωt )
9π 2mL
2 2
3L
The hard way is as follows
+∞ +∞
d
< p x >= ∫ Ψ ∗ ( x, t )( px )op Ψ ( x, t )dx = −ih ∫ Ψ ∗ ( x, t ) Ψ ( x, t )dx
−∞ −∞
dx
+∞
 1 1 ∗ d  1 1 
= −ih ∫  ψ 1∗ ( x)e + iω1t + ψ 2 ( x)e + iω 2 t   ψ 1 ( x)e − iω1t + ψ 2 ( x)e − iω 2 t dx
−∞  2 2  dx  2 2 
+∞ +∞
− ih dψ ( x ) − ih dψ ( x )
=
2 −∞∫ ψ 1∗ ( x) 1 dx +
dx 2 −∞∫ ψ 2∗ ( x) 2 dx
dx
+∞ +∞
− ih + iω1t − iω 2 t dψ ( x ) ih dψ ( x )
=
2
e e ∫
−∞
ψ 1∗ ( x) 2 dx − e + iω 2 t e − iω1t ∫ ψ 2∗ ( x) 1 dx
dx 2 −∞
dx
ih + iω1t − iω 2 t  8  ih + iω 2 t − iω1t  8 
= 12 < p x >1 + 12 < p x > 2 − e e − − e e  
2  3L  2  3L 
=
3L
e (
ih 4 + i∆ωt
− e − i∆ωt ) 8h
= − sin( ∆ωt )
3L
where I used
+∞
dψ 2 ( x ) 2 2π
L

∫ 1
ψ
L L ∫0
sin(πx / L) cos(2πx / L)dx

( x ) dx =
−∞
dx
1 2π
L
=
L L ∫ (sin(3πx / L) − sin(πx / L))dx
0

1 2π 1 2π
L L
4 1  8
=
L L 0∫ sin(3πx / L)dx −
L L 0∫ sin(πx / L)dx =  − 1 = −
L3  3L

Department of Physics Page 5 of 12 Exam 1 Solutions


PHY4604 Fall 2006 R. D. Field

and
L 3π
1 1 L 1 3π 2
L0∫ sin(3πx / L)dx = ∫
L 3π 0
sin( y )dy = − (cos y ) 0 =
3π 3π
L π
1 1L 1 π 2
L0∫ sin(πx / L)dx = ∫
Lπ 0
sin( y )dy = − (cos y ) 0 =
π π
and
+∞
dψ ( x ) 2π
L

∫− ∞ψ ( x) dx1 dx = L L ∫0 sin(2πx / L) cos(πx / L)dx



2


L
(sin(3πx / L) + sin(πx / L) )dx
L L ∫0
=

1π 1π
L L
2 1  8
=
LL0 ∫ sin(3πx / L)dx +
LL0 ∫ sin(πx / L)dx =  + 1 =
L  3  3L
(i) (6 points) Suppose the electron in this infinite square well has a wave function at t = 0 which
is given by
2
Ψ ( x,0) = sin(πx / L) cos(3πx / L) .
L
If you measure the energy of this particle, what are the possible values you might get, and what
is the probability of getting each of them? What is the expectation value of the energy for this
state (i.e. average energy)?
π 2h 2
Answers: E2 with probability ½ and energy E4 with probability ½, < E >= 10 E0 = 5
mL2
Solution: We see that
2 1 1
Ψ ( x ,0 ) = sin(πx / L) cos(3πx / L) = sin( 4πx / L) − sin( 2πx / L)
L L L
1 2 1 2 1 1
= sin( 4πx / L) − sin( 2πx / L) = ψ 4 ( x) − ψ 2 ( x)
2 L 2 L 2 2
This state is thus a mixture of ψ 2 and ψ 4 with c2 = - 1 / 2 and c4 = 1 / 2 and all the other ci’s
are zero. This means that you get energy E2 with probability ½ and energy E4 with probability
½. The expectation value of E is
1 π 2h 2
< E >= 12 E2 + 12 E4 = (4 E0 + 16 E0 ) = 10 E0 = 5
2 mL2
Problem 3 (25 points): Suppose that particles with energy E > V0 enter from Delta-Function + Step
the left and travel to the right and encounter both a delta-function potential and E

a step-function potential at x = 0 as follows: V0

 0 x<0
V ( x) = 
V0 − αδ ( x) x ≥ 0 x=0 x

where V0 and α are positive (real) constants. Let


-αδ(x)
V 2mα h2 α 2m
r = 1− 0 and R= 2 = .
E h 2mE h E

Department of Physics Page 6 of 12 Exam 1 Solutions


PHY4604 Fall 2006 R. D. Field

(a) (5 points) Calculate the quantum mechanical reflection probability, PR, and express your
answer in terms of r and R.
(1 − r ) 2 + R 2
Answer: PR =
(1 + r ) 2 + R 2
Solution: We look for solutions of the time-independent Schrödinger equation
h 2 d 2ψ ( x ) d 2ψ ( x ) 2m
− 2
+ V ( x )ψ ( x ) = E ψ ( x ) or 2
= − 2 ( E − V ( x ))ψ ( x )
2m dx dx h
with Ψ ( x, t ) = ψ ( x)e − iEt / h
. In the region x < 0 (left region) for E > 0 and V(x) = 0 we have
d 2ψ ( x ) 2mE 2mE h 2k 2
= − 2 ψ ( x ) = − k 2ψ ( x ) with k = and E =
dx 2
h h2 2m
The most general solution is
ψ L ( x ) = AL e + ikx + BL e − ikx ,
In the region x > 0 (right region) we have

ψ R ( x ) = AR e + iqx + BR e − iqx , where q = 2m( E 2− V0 ) = rk


h
Since there are no particles entering from the right in the “right” region we set BR = 0. The
boundary conditions at x = 0 are
ψ L (0) = ψ R (0) which implies that (1) AL + BL = AR.
Also,
dψ R ( x ) dψ L ( x ) 2 mα
− = − 2 ψ R ( 0)
dx x=+ ε dx x =− ε h
which implies
2 mα q 2 mα 
iqAR − ikAL + ikBL = − AR or (2) AL − BL =  − i  AR = (r − iR )AR
h 2
k kh 2 
Adding (1) + (2) yields
2 AL
2 AL = (1 + r − iR )AR which implies AR =
(1 + r − iR )
And subtracting (1)-(2) gives

2 B L = (1 − r + iR )AR which implies B L = 1


(1 − r + iR )AR = (1 − r + iR )AR .
2
(1 + r − iR )
The reflection probability is
hk
| B L | 2 (1 − r ) 2 + R 2
PR = m
= .
hk
m | AL | 2 (1 + r ) 2 + R 2

(b) (5 points) Calculate the quantum mechanical transmission probability, PT, and express your
answer in terms of r and R.

Department of Physics Page 7 of 12 Exam 1 Solutions


PHY4604 Fall 2006 R. D. Field

4r
Answer: PT =
(1 + r ) 2 + R 2
Solution: The transmission probability is
hq
| AL |2 4r
PT = m
= .
hk
m
| AL | 2
(1 + r ) 2 + R 2

(c) (2 points) Show that PT + PR = 1.


Solution: Adding PR and PT gives
(1 − r ) 2 + R 2 4r (1 − r ) 2 + 4 r + R 2 (1 + r ) 2 + R 2
PR + PT = + = = = 1.
(1 + r ) 2 + R 2 (1 + r ) 2 + R 2 (1 + r ) 2 + R 2 (1 + r ) 2 + R 2
(d) (4 points) What is the transmission probability, PT1, for the case R = 0 (i.e. no delta
function)? What is the numerical value of PT1 for the case V0/E = 3/4 (with R = 0)?
4r 8
Answer: PT 1 = r→ ≈ 0.889
=1 / 2
(1 + r ) 2
9
Solution: For R = 0 the transmission probability becomes
4r 2 8
PT 1 = r→
=1 / 2
= ≈ 0.889 .
(1 + r ) 2
(1 + 1 / 2) 2
9
Note that V0/E = 3/4 implies that r = ½.
(e) (4 points) What is the transmission probability, PT2, for the case V0 = 0 (i.e. no step)?
What is the numerical value of PT2 for the case R = 2 (with V0 = 0)?
4 1
Answer: PT 2 = R

=2

4+ R 2
2
Solution: For r = 0 the transmission probability becomes
4 4 1
PT 2 =  
 → = .
4 + R2 R=2 4 + 4 2
Note that V0 = 0 implies that r = 1.
(f) (5 points) What is the the numerical value of overall transmission probability, PT, for the case
V0/E = 3/4 and R = 2? Does PT = PT1PT2?
8 4
Answers: PT == ≈ 0.32 , PT 1 PT 2 = ≈ 0.44 ≠ PT .
25 9
Solution: For r = ½ and R = 2 the transmission probability becomes
4r 2 8
PT =     → = ≈ 0.32 .
(1 + r ) 2 + R 2 r =1 / 2 , R = 2 9 / 4 + 4 25
We that
 8  1  4
PT 1 PT 2 =    = ≈ 0.44 ≠ PT .
 9  2  9

Department of Physics Page 8 of 12 Exam 1 Solutions


PHY4604 Fall 2006 R. D. Field

Problem 4 (25 points): Consider the one dimensional harmonic oscillator with potential energy
1
V ( x) = Kx 2 . The Hamiltonian is given by
2
p 2 mω 2 x 2
p2
H = x +V =
2m
1
2m
(
px2 + (mωx) 2 = x +
2m
)
2
where ω = K / m . We express the Hamiltonian in terms of the operators (px)op and (x)op, as
follows
H op =
1
2m
(
( p x2 ) op + m 2ω 2 ( x 2 ) op , )
and we define the two operators (a± )op as follows

(a± ) op ≡
1
(mωxop m i( px )op ).
2hmω
(a) (2 points) Given that [( px )op , ( x) op ] = −ih , prove that [(a-)op,(a+)op] = 1.
Solution: We see that
1
[(a− ) op , (a+ ) op ] = [mωxop + i ( p x ) op , mωxop − i ( p x ) op ]
2hmω
=
1
2hmω
( )
m 2ω 2 [ xop , xop ] − imω[ xop , ( p x ) op ] + imω[( p x ) op , xop ] + [( p x ) op , ( p x ) op ]

i
= [( p x ) op , xop ] = 1
h
(b) (2 points) Prove that H op = hω ((a+ ) op (a− )op + 12 ) = hω ((a− )op (a+ )op − 12 ) .
Solution: We see that
1
(a+ )op (a− )op = (mωxop − i( px )op )(mωxop + i( px )op )
2hmω
1  ( px ) op 1 
2

=
1
2hmω
(
m 2ω 2 xop
2
+ ( p x )op
2
− imω[( p x )op , xop ] = )
hω  2m
+
2
mω 2 2
xop −
1
2
h ω 


1 1
= H op −
hω 2
and hence
H op = hω ((a+ ) op (a− )op + 12 )
Also,
H op = hω ((a+ ) op (a− ) op + 12 ) = hω ((a− ) op (a+ ) op + [(a+ ) op , (a− ) op ] + 12 )
= hω ((a− ) op (a+ ) op − 12 )
(c) (2 points) Show that [ H op , (a± )op ] = ±hω (a± )op .
Solution: We see that
[ H op , (a± )op ] = hω[(a+ ) op (a− )op + 12 , (a± )op ] = hω[(a+ ) op (a− )op , (a± )op ]
= hω (a+ )op [(a− )op , (a± )op ] + hω[(a+ ) op , (a± )op ](a− )op = ± hω (a± ) op
where I used [AB,C]=A[B,C]+[A,C]B.
(d) (5 points) Consider the eigenvalue equation

Department of Physics Page 9 of 12 Exam 1 Solutions


PHY4604 Fall 2006 R. D. Field

H op | n >= En | n > , where En = (n + 12 )hω , with n = 0, 1, 2, …


Using Dirac notation show that the eigen kets |n> form an orthonormal set (i.e. show that <n|m>
= δnm) and show that Nop|n> = n|n>, where N op = (a+ )op (a− )op (i.e. |n> are eigenkets of the Nop
with eigenvalue n). Using Dirac notation show that (a+ )op | n >= n + 1 | n + 1 > and
(a− ) op | n >= n | n − 1 > .
Solution: First we normalize <n|n> = 1. Now suppose that H op | n >= En | n > and
H op | m >= Em | m > for n ≠ m. Then operating with Hop to the right yields
(1) < n | H op | m >=< n | Em | m >= Em < n | m > and operating with Hop to the left yields
(2) < n | H op | m >=< n | En | m >= En < n | m > , where I used
( H op | n >)↑ =< n | H op ↑
=< n | H op =< n | En∗ =< n | En .
and I used the fact that Hop is hermitian and hence En is real. Subtraction (2) from (1) gives
0 = ( Em − En ) < n | m > and since En ≠ Em, then <n|m> = 0. From part (b) we see that
N op = (a+ ) op (a− )op = h1ω H op − 12 and (a− )op (a+ )op = h1ω H op + 12
and hence
N op | n >= (h1ω H op − 12 ) | n >= ( h1ω En − 12 ) | n >= n | n > ,
where I used h1ω En = n + 12 . Now lets look at the states (a±)op|n> = |a±n>. We see that
Hop|a±n> = Hop(a±)op |n> = (a±)opHop |n> + [Hop,(a±)op] |n>= (En ± hω )|a±n>
and hence
(a+ )op | n >= cn | n + 1 > and (a− )op | n >= d n | n − 1 > ,
where cn and dn are constants, but
< n | (a− )op (a+ )op | n >=| cn |2 and < n | (a+ )op (a− )op | n >=| d n |2
and hence
| cn |2 =< n | (a− )op (a+ )op | n >=< n | (h1ω H op + 12 ) | n >= ( h1ω En + 12 ) < n | n >= (n + 1)
| d n |2 =< n | N op | n >= n < n | n >= n
Taking cn and dn to be real gives
cn = n + 1 and d n = n .
(e) (5 points) Using operator methods and the Dirac notation from (d), and the fact that

( x) op =
h
((a+ )op + (a− )op ) ( px )op = i hmw ((a+ )op − (a− )op )
2mω 2
evaluate <x>n = <n|(x)op|n> and <px> = <n|(px)op|n> for the nth stationary state.
Answers: <x>n = 0 and <px> = 0
Solution: We see that

Department of Physics Page 10 of 12 Exam 1 Solutions


PHY4604 Fall 2006 R. D. Field

h
< x > n =< n | ( x) op | n >= < n | (a+ ) op + (a− ) op | n >
2mω

=
h
(< n | (a+ )op | n > + < n | (a− )op | n >)
2mω

2mω
h
= (
n +1 < n | n +1 > + n < n | n −1 > = 0 )
We can calculate <px>n from <x>n as follows:
d < x >n
< px > n = m =0
dt
or we can calculate it the long way as follows
hmω
< px > n =< n | ( px )op | n >= i < n | (a+ )op − (a− ) op | n >
2
hmω
=i (< n | (a+ )op | n > − < n | (a− )op | n >)
2
hmω
2
=i (
n +1 < n | n +1 > − n < n | n −1 > = 0 )
(f) (9 points) Suppose that a particle in the harmonic oscillator potential at t = 0 is in the state
given by
| Ψ (0) >= A(| 0 > +2 | 1 > ) ,
where |0> and |1> are the first two stationary states from part (e). What is the normalization
constant A? If you measure the energy of this particle, what are the possible values you might
get, and what is the probability of getting each of them? What is the expectation value of the
energy for this state (i.e. average energy)? At later times, what is | Ψ (t ) > ? What is <x> and
<px> for the state | Ψ (t ) > ? Do <x> and <px> depend on time?
Answers: E0 with probability 1/5 and E1 with probability 4/5, < E >= 10 13
hω ,
1 2 4 h 4 hmω
| Ψ (t ) >= | 0 > e − i ωt / 2 + | 1 > e − 3iωt / 2 , < x >= cos(ωt ) , < p x >= − sin(ωt ) ,
5 5 5 2mω 5 2
yes they depend on time.
Solution: The normalization A is determined by requiring that
< Ψ (0) | Ψ (0) >= 1 = A2 (2 < 1 | + < 0 |)(| 0 > +2 | 1 > ) = 5 A2
Thus, A = 1 / 5 and
1 2 1 2
| 0 > e − iE0 t / h +
| Ψ (t ) >= | 1 > e − iE1t / h = | 0 > e − i ωt / 2 + | 1 > e − 3 i ωt / 2 .
5 5 5 5
which means that you get energy E0 with probability 1/5, and energy E1 with probability 4/5.
The expectation value of E is
< E >= 15 E0 + 54 E1 = ( 15 ( 12 ) + 54 ( 32 ) )hω = 1013
hω .

Department of Physics Page 11 of 12 Exam 1 Solutions


PHY4604 Fall 2006 R. D. Field

h
< x >=< Ψ (t ) | ( x)op | Ψ (t ) >= < Ψ (t ) | (a+ )op + (a− )op | Ψ (t ) >
2mω
h  2   1 
< 0 | e + iωt / 2 ((a+ )op + (a− )op )
1 2
=  < 1 | e + 3 i ωt / 2 + | 0 > e − iωt / 2 + | 1 > e − 3 i ωt / 2 
2mω  5 5   5 5 
h 1 2 + iωt / 2 − 3iωt / 2 h 2 1 + 3iωt / 2 − iωt / 2
= e e < 0 | (a− )op | 1 > + e e < 1 | (a+ )op | 0 >
2mω 5 5 2mω 5 5

=
h 2 − i ωt
2mω 5
(
e + e + i ωt = )
4
5 2mω
h
cos(ωt )

We can calculate <px> from <x> as follows:


d<x> 4 h 4 hmω
< p x >= m = −mω sin(ωt ) = − sin(ωt ) ,
dt 5 2mω 5 2
or we can do it the hard way
hmω
< px >=< Ψ (t ) | ( px )op | Ψ (t ) >= i < Ψ (t ) | (a+ )op − (a− )op | Ψ (t ) >
2
hmω  2   1 
< 0 | e + iωt / 2 ((a+ )op − (a− )op )
1 2
=i  < 1 | e + 3iωt / 2 + | 0 > e − iωt / 2 + | 1 > e − 3 i ωt / 2 
2  5 5   5 5 
hmω 1 2 + iωt / 2 − 3iωt / 2 hmω 2 1 + 3iωt / 2 − iωt / 2
= −i e e < 0 | (a− )op | 1 > +i e e < 1 | (a+ )op | 0 >
2 5 5 2 5 5
hmω 2 − iωt 4 hmω
= −i
2 5
(
e − e + iωt = −
5 2
)
sin(ωt )

Department of Physics Page 12 of 12 Exam 1 Solutions

S-ar putea să vă placă și